Triangle UVW, with vertices U(-5,5), V(-4,7), and W(-9,8), is drawn on the coordinate grid below.

Triangle UVW, With Vertices U(-5,5), V(-4,7), And W(-9,8), Is Drawn On The Coordinate Grid Below.

Answers

Answer 1

The area formula of a triangle given the coordinates of the vertices :

[tex]U(-5,5),V(-4,7),W(-9,8)[/tex][tex]A=\lvert\frac{U_x(V_y-W_y)+V_x(W_y-U_y)+W_x(U_y-V_y)}{2}\rvert[/tex]

Using the formula above, the area will be :

[tex]\begin{gathered} A=\lvert\frac{-5(7-8)-4(8-5)-9(5-7)}{2}\rvert \\ A=\lvert\frac{5-12+18}{2}\rvert \\ A=\lvert\frac{11}{2}\rvert \\ A=\lvert5.5\rvert \\ A=5.5 \end{gathered}[/tex]

The answer is 5.5 square units


Related Questions

Hi there, I need help with this question. Thank you in advance!

Answers

For the data given, we have 24 entries in all.

They are :

75, 36, 80, 49, 24, 61, 34, 39, 30, 76, 44, 44, 40, 35, 21, 89, 34, 70, 79, 65, 66, 53, 99, 11

(1) Minimum refers to the lowest data in the table. we can see out of all the data in the table, the lowest is 11. Therefore,

Min = 11

(2) Maximum refers to the highest data in the table.

The highest is 99.

Therefore,

Max = 99

(3) Range is defined as highest data minus lowest data

Range = 99 - 11

Range = 88

(4) Mean:

[tex]\begin{gathered} \text{ Mean =}\frac{\text{ sum of the data}}{total\text{ count}} \\ \operatorname{mean}\text{ = }\frac{75+36+80+49+24+61+34+39+30+76+44+44+40+35+21+89+34+70+79+65+66+53+99+11}{24} \\ \\ \text{Mean = }\frac{1254}{24} \\ =52.25 \end{gathered}[/tex]

Therefore,

Mean = 52.25

(5) Standard deviation:

The steps to calculate the standard deviation in shown in the picture below.

The standard deviation = 22.8386..

To 2 decimal places, we have 22.84

Therefore,

Standard deviation = 22.84

What is the answer to 3/8 + 7 5/8

Answers

Given the Addition:

[tex]\frac{3}{8}+7\frac{5}{8}[/tex]

You can find the sum as follows:

1. Covert the Mixed Number to an Improper Fraction:

- Multiply the Whole Number part by the denominator of the fraction.

- Add the result to the numerator.

- The denominator does not change.

Then:

[tex]7\frac{5}{8}=\frac{5+(7\cdot8)}{8}=\frac{5+56}{8}=\frac{61}{8}[/tex]

2. Rewrite the Addition:

[tex]=\frac{3}{8}+\frac{61}{8}[/tex]

3. Since the denominators are equal, you only need to add the numerators:

[tex]=\frac{3+61}{8}=\frac{64}{8}[/tex]

4. Simplifying the fraction, you get:

[tex]=8[/tex]

Hence, the answer is:

[tex]=8[/tex]

are f(x) and g(x) inverse functions across the domain (5, + infinity)

Answers

Given:

[tex]\begin{gathered} F(x)=\sqrt{x-5}+4 \\ G(x)=(x-4)^2+5 \end{gathered}[/tex]

Required:

Find F(x) and G(x) are inverse functions or not.

Explanation:

Given that

[tex]\begin{gathered} F(x)=\sqrt{x-5}+4 \\ G(x)=(x-4)^{2}+5 \end{gathered}[/tex]

Let

[tex]F(x)=y[/tex][tex]\begin{gathered} y=\sqrt{x-5}+4 \\ y-4=\sqrt{x-5} \end{gathered}[/tex]

Take the square on both sides.

[tex](y-4)^2=x-5[/tex]

Interchange x and y as:

[tex]\begin{gathered} (x-4)^2=y-5 \\ y=(x-4)^2+5 \end{gathered}[/tex]

Substitute y = G(x)

[tex]G(x)=(x-4)^2+5[/tex]

This is the G(x) function.

So F(x) and G(x) are inverse functions.

[tex]\begin{gathered} G(x)-5=(x-4)^2 \\ \sqrt{G(x)-5}=x-4 \\ x=\sqrt{G(x)-5}+4 \end{gathered}[/tex]

Final Answer:

Option A is the correct answer.

A ball is thrown from an initial height of 6 feet with an initial upward velocity of 21 ft/s. The ball's heighth (in feet) after t seconds is given by the following,6+21 167Find all values of t for which the ball's height is 12 feet.Round your answer(s) to the hearest hundredth(If there is more than one answer, use the "or" button.)

Answers

The given expression in the question is

[tex]h=6+21t-16t^2[/tex]

with the value of h given as

[tex]h=12ft[/tex]

By equating both equations, we will have

[tex]\begin{gathered} 12=6+21t-16t^2 \\ 12-6-21t+16t^2=0 \\ 6-21t+16t^2=0 \\ 16t^2-21t+6=0 \end{gathered}[/tex]

To find the value of t we will use the quadratic formula of

[tex]ax^2+bx+c=0[/tex]

which is

[tex]\begin{gathered} x=\frac{-b\pm\sqrt[]{b^2-4ac}}{2a} \\ \text{where} \\ a=16 \\ b=-21 \\ c=6 \end{gathered}[/tex]

By substitution, we will have

[tex]\begin{gathered} t=\frac{-(-21)\pm\sqrt[]{(-21)^2-4\times16\times6}}{2\times16} \\ t=\frac{21\pm\sqrt[]{441-384}}{32} \\ t=\frac{21\pm\sqrt[]{57}}{32} \\ t=\frac{21\pm7.5498}{32} \\ t=\frac{21+7.5498}{32}\text{ or t=}\frac{21-7.5498}{32} \\ t=\frac{28.5498}{32\text{ }}\text{ or }t=\frac{13.4502}{32\text{ }} \\ t=0.89\text{ or t=0.42} \end{gathered}[/tex]

Alternatively, Solving the equation graphically we will have

Therefore,

The values of t( to the nearest hundredth) t= 0.89sec or 0.42sec

andrews family spent 410 on 2 adult tickets to go to the concert. maxs family spent 375 on 3 tickets and 2 child tickets 3 how much is the adult ticket how much is a child ticket

Answers

Let 'x' represents the cost of the ticket for an adult, and 'y' be the cost of a child's ticket.

Andrew's family spend 410 on two adult tickets.

[tex]2x=410[/tex]

From the above expression,

[tex]\begin{gathered} x=\frac{410}{2} \\ =205 \end{gathered}[/tex]

Thus, the cost of an adult ticket is 205.

Given that the Max's family spend 375 for 3 tickes, and two of them is for childrens.

[tex]\begin{gathered} x+2y=375 \\ 205+2y=375 \\ 2y=375-205 \\ 2y=170 \\ y=\frac{170}{2} \\ y=85 \end{gathered}[/tex]

Thus, the cost for tickets for the childrens is 85.

What is the distance from 7 to 0? O A. 7, because 171 = 7 Jurid O B. 7, because 171 = 7 O c. 7, because |-71 = -7 O D. -7, because [7] = -7

Answers

The distance from 7 to 0 is 7 because the absolute value of 7 is 7.

Correct Answer: A

Greg is ordering tile for a floor he is installing. The owner picks out tile that is 16in by 16in including the grout . The floor is 350 sq ft . (part 1) How many tile must Greg order for the floor ( assume no waste)(part 2) Each tile cost $ 1.75 plus 8% sales tax . How will the tile cost ?

Answers

ANSWER

(part 1) 196 tiles

(part 2) $ 1.89

EXPLANATION

(part 1)

First we have to find the area of each tile, that is the product of the dimensions because it is a rectangle,

[tex]A_{\text{tile}}=16in\cdot16in=256in^2[/tex]

To compare it to the floor's area, we have to transform it into square feet. Knowing that 1 ft² = 144 in²,

[tex]256in^2\cdot\frac{1ft^2}{144in^2}=\frac{16}{9}ft^2[/tex]

This is a partial result, so it is best if we leave it as a fraction so we don't miss any decimals.

Now, the area of the floor is 350 ft². To find how many tiles Greg has to order, we have to divide the area of the floor by the area of each tile,

[tex]\#tiles=\frac{A_{\text{floor}}}{A_{\text{tile}}}=\frac{350ft^2}{\frac{16}{9}ft^2}=196.875[/tex]

But the number of tiles has to be an integer. If Greg buys 197 tiles they will have to cut some (waste). If he buys 196 there will be some of the floor not covered. However we were asked to assume no waste, so Greg will have to order 196 tiles.

(part 2)

To answer this question we have to add 8% to the cost of the tile. The 8% of 1.75 is,

[tex]1.75\cdot\frac{8}{100}=0.14[/tex]

So the cost of each tile is,

[tex]1.75+0.14=1.89[/tex]

The smaller of two similar balloons has a diameter of 10 inches. If it takes 12 (same sized) breaths to blow up the smaller balloon and 40.5 to blow up the larger, what is the diameter of the larger balloon?

Answers

The smaller of two similar balloons has a diameter of 10 inches. If it takes 12 (same sized) breaths to blow up the smaller balloon and 40.5 to blow up the larger, what is the diameter of the larger balloon?​

we have that

If two figures are similar, then the ratio of its volumes is equal to the scale factor elevated to the cube.

so

Find the scale factor

ratio volumes=40.5/12=3.375

3.375=(scale factor)^3

[tex]\text{scale factor=}\sqrt[3]{3.375}[/tex]

scale factor=1.5

To find out the diameter of the larger balloon multiply the scale factor by the diameter of the smaller balloon

so

1.5*(10)=15 inches

the answer is 15 inches

I wills send you a picture

Answers

Draw the tank

we can use the formula of the volume of a cylinder

[tex]V=\pi\times r^2\times h[/tex]

we can repalce the value of the volume (320pi) and the height

[tex]\begin{gathered} 320\pi=\pi\times r^2\times20 \\ 320\pi=20r^2\pi \end{gathered}[/tex]

now solve for r^2 dividing 20pi on both sides

[tex]\begin{gathered} \frac{320\pi}{20\pi}=r^2 \\ \\ r^2=16 \\ \end{gathered}[/tex]

and solve for r using roots

[tex]\begin{gathered} r=\sqrt[]{16} \\ \\ r=4 \end{gathered}[/tex]

the value of the radious is 4ft and the diameter double, then

[tex]\begin{gathered} d=2\times4 \\ d=8 \end{gathered}[/tex]

diameter of the cylinder is 8 ft then rigth option is C

4) Using the number line to help you, decide which fraction is larger or if they are equal: one/twos or three/fifths. Label each fraction on the number line.

Answers

Explanation:

The number line is between 0 to 1. There are 10 smaller lines in between

Each of the small lines represent 1/10 or 0.1

one/twos is the same as 1/2 = 0.5

three/fifths is the same as 3/5 = 0.6

From the above, 0.6 is greater than 0.5

Showing both numbers on the number line:

11. Jarrod's favorite movie is now on video at 15% off the originalprice. If he pays $17, what was the original price?

Answers

original price = x

discount = 15%

price after discount = $17

Write an equation:

x (1-15/100) = 17

x (1-0.15 ) = 17

0.85x = 17

Solve for x:

x = 17/0.85

x = $20

How do I find the selling price if a store pays 3$ for a magazine. The markup is 5%

Answers

We need to find the selling price of a magazine. We know that the store pays $3 for it, and the markup is 5%.

So, we need to add 5% of the initial price to that initial price.

First, let's find:

[tex]5\%\text{ of }\$3=5\%\cdot\$3=\frac{5}{100}\cdot\$3=\frac{\$15}{100}=\$0.15[/tex]

Now, adding the previous result to the initial price, we obtain:

[tex]\$3+\$0.15=\$3.15[/tex]

Therefore, the selling price is $3.15.

i need help with this question

Answers

Answer:

8%.

Step-by-step explanation:

The perimeter = 2(20 + 30)

                         = 100 cm.

The new perimeter = 2(20 + 0.05*20 + 30 + 30*0.10)

                                 = 2(21 + 33)

                                 = 2*54

                                 = 108 cm.

Percent increases = 8%.

Kepler's third law of planetary motion states that the square of the time required for a planet to make one revolution about the sun varies directly as the cube of the average distance of the planet from the sun. If you assume that Jupiter is 5.2 times as far from the sun as is the earth, find the approximate revolution time for Jupiter in years.

Show work pls ;-;

Answers

By applying Kepler's third law of planetary motion, the approximate revolution time for Jupiter is equal to 12 years.

What is Kepler's third law?

Mathematically, Kepler's third law of planetary motion is given by this mathematical expression:

T² = a³

Where:

T represents the orbital period.a represents the semi-major axis.

Note: Earth has 1 astronomical unit (AU) in 1 year of time.

For this direct variation, the value of the constant of proportionality (k) is given by:

T² = ka³

k = T²/a³

k = 1²/1³

k = 1.

When the semi-major axis or the distance of Jupiter from Sun is 5.2, we have;

T² = ka³

T² = 1 × 5.2³

T² = 140.608

T = √140.608

T = 11.858 ≈ 12 years.

Read more on Kepler's third law here: https://brainly.com/question/15691974

#SPJ1

Consider the two polynomials p(x), q(x) in Z[x] by p(x) = 1+2x+3x2, q(x) = 4+5x+7x3. Then p(x) + q(x) is

Answers

The solution for polynomials p(x) + q(x) is 7x³ + 3x² + 7x + 5

Given,

The polynomials

p(x) = 1 + 2x + 3x²

q(x) = 4 + 5x + 7x³

We have to find the solution for p(x) + q(x)

Then,

p(x) + q(x) = (1 + 2x + 3x²) + ( 4 + 5x + 7x³)

p(x) + q(x) = 7x³ + 3x² + 2x + 5x + 1 + 5

p(x) + q(x) = 7x³ + 3x² + 7x + 5

That is,

The solution for polynomials p(x) + q(x) is 7x³ + 3x² + 7x + 5

Learn more about polynomials here:

https://brainly.com/question/13055309

#SPJ1

Evaluate theexpression belowwhen x = = 3.<54 : 2.3 - 22Enter your answer inthe box below.

Answers

The given expression is

[tex]54\frac{.}{.}2\times3-x^2[/tex]

where x=3

the dot in the expression means multiplication

substitute into the expression above we have

[tex]\begin{gathered} 54\frac{.}{.}2\times3-3^2 \\ \end{gathered}[/tex]

Applying BODMAS

[tex]27\times3-3^2[/tex][tex]\begin{gathered} 81-3^2 \\ 81-9 \\ 72 \end{gathered}[/tex]

Therefore the value of the expression is 72

The graphs of the functions g and h are shown below. For each graph, find the absolute maximum and absolute minimum. If no such value exists, click on "None".

Assume that the dashed line shown is a vertical asymptote that the graph does not cross.

Answers

For the graph g, the absolute maximum is 2 and the absolute minimum is -4.

Similarly, for graph h, the absolute maximum is 3 and the absolute minimum is -5.

Absolute Maximum of a Graph:

The absolute maximum of a graph is the point on the graph with the highest y-value. There can only be one absolute maximum of a graph.

Absolute Minimum of a Graph:

The absolute minimum of a graph is the point on the graph with the lowest y-value. There can only be one absolute minimum of a graph.

Given,

Here we have the two graph called g and h.

Now, we need to find the absolute maximum and minimum from it.

AS per the given definition, we know that,

For graph g,

The absolute maximum is 2 and the absolute minimum is -4.

Similarly, for graph h, the absolute maximum is 3 and the absolute minimum is -5.

To know more about Absolute maximum and minimum here.

https://brainly.com/question/28767824

#SPJ1

what is the simplified ratio of 32:24

Answers

Answer:

4/3

Step-by-step explanation:

The simplest form of

32: 24

is 43

Steps to simplifying fractions

Find the GCD (or HCF) of numerator and denominator

GCD of 32 and 24 is 8

Divide both the numerator and denominator by the GCD

32 ÷ 8

24 ÷ 8

Reduced fraction:

4/3

Therefore, 32/24 simplified to lowest terms is 4/3.

Angie added a stone border 2 feet in width on all sides of her garden making her harder 12 by 6 feet. What is the area, in square feet, of the portion of the garden that excludes the border?

A. 4
B. 16
C. 40
D. 56
E. 72

Answers

The area, in square feet, of the portion of the garden that excludes the border is 40.

What is the area of the rectangle?

The area of the rectangle is the product of the length and width of a given rectangle.

The area of the rectangle = length × Width

We have been given that Angie added a stone border of 2 feet in width on all sides of her garden making her harder 12 by 6 feet.

Length = 12 ft

Width = 6 ft

The dimension of the garden  that excludes the border of 2 feet are;

Length = 12 ft- 2 = 10 ft

Width = 6 ft - 2= 4 ft

Thus, Area  =  length × Width

Area = 10 x 4

Area = 40 square feet

Hence, the area, in square feet, of the portion of the garden that excludes the border is 40.

Learn more about the area;

https://brainly.com/question/1658516

#SPJ1

Choose SSS, SAS, or neither to comparethese two triangles.A) SSSB) SASC) neither

Answers

Answer:

C. Neither

Explanation:

The SSS Congruence Rule states that if the three sides of a triangle are equal to the three sides of another triangle, then the two triangles are congruent.

The SAS Congruence Rule states that if the two sides and the included angle of one triangle are equal to the two sides and the included angle of another triangle, then the two triangles are congruent.

Notice that in the given triangles, there are two congruent sides and a non-included angle, since this does not satisfy any of the rules stated above, SSS Congruence rule or SAS Congruence rule, we'll choose "neither" as the correct answer.

Function g is a transformation of the parent function exponential function. Which statements are true about function g?

Answers

For the given function, The following are true statements:

Four units separate function g from function f.There is a y-intercept for function g. (0,4)Function g has a range of (3,∞ ).Over the range (-, ∞), function g is positive.

It may be seen from the graph below that

The g function's graph is 4 units higher than the parent exponential function's graph.

All of the input values for which the function is defined are referred to as the function's domain. The domain of the function is (-, ∞ ) according to the graph of function g.

The location where a function's graph crosses the y-axis is known as the y-intercept. G's graph crosses the y-axis at (0, 4). As a result, the Function g's y-intercept is (0,4).

growing function g across the range (- ∞, 0).

Function output values are referred to as the function's range. It can be seen from the graph that the range of function g is (3, ∞ ).

To learn more about functions, https://brainly.com/question/21145944

#SPJ9

Nov 28,Quadrilateral ABCD is dilated by a scale factor of to form quadrilateral A'B'C'D'.What is the measure of side DA?Dal1515301B2162А

Answers

The quadrilateral ABCD was dilated by a scale factor of 3/4 to form the quadrilateral A'B'C'D'

This means that each side of the quadrilateral was multiplied by 3/4 to make the dilation.

You know that the scale factor is 3/4 and the length of D'A' is equal to 30 units, then:

[tex]\begin{gathered} D^{\prime}A^{\prime}=\frac{3}{4}DA \\ 30=\frac{3}{4}DA \end{gathered}[/tex]

Multiply both sides by the reciprocal of 3/4

[tex]\begin{gathered} 30\cdot\frac{4}{3}=(\frac{4}{3}\cdot\frac{3}{4})DA \\ 40=DA \end{gathered}[/tex]

The length of side DA is 40 units.

Hey I need help with my homework help me find the points on the graph too please Thankyouu

Answers

Given the function:

g(x) = 3^x + 1

we are asked to plot the graph of the function.

Using the table:

x y

-2 10/9

-1 4/3

0 2

1 4

2 10

The graph:

The expomential functions have a horizontal asymptote.

The equation of the horizontal asymptote is y = 1

Horizontal Asymptote: y = 1

To find the domain is finding where the question is defined.

The range is the set of values that correspond with the domain.

Domain: (-infinity, infinity), {x|x E R}

Range: (1, infinity0, {y|y > 1}.

2) A seat has a speed of 15 mph in seicher travels downstream from Greentown to Glenevon in 2/5 of an hour. It then goes back upstream from Glenevon to Colombia, which is 2 miles downstream, in 3/5 of an hour. Find the rate of the current.

Answers

Answer:

Rate of the current = 5 mph

Explanations:

The time taken to travel downstream from Greentown to Glenevon = 2/5 hours

The time taken to travel upstream from Glenevon to Columbia = 3/5 hours

Rate = Distance /time

Let the distance traveled = y miles

Rate of the downstream travel:

Rate = y ÷ 2/5

Rate = y x 5/2

Rate = 5y / 2

Rate of the upstream travel

Since the travel upstream from Glenevon to Columbia is 2 meters downstream:

Distance = y - 2

Rate = (y-2) ÷ 3/5

Rate = (y-2) x 5/3

Rate = 5(y-2)/3

Let the current rate be represented by k

The rate of the downstream travel will be:

15 + k = 5y/2...........(1)

The rate of the upstream movement will be:

15 - k = 5(y-2)/3............(2)

Add equations (1) and (2)

(15 + k) + (15 - k) = [5y/3] + [5(y-2)/3]

[tex]\begin{gathered} 30\text{ = }\frac{5y}{2}+\text{ }\frac{5y-10}{3} \\ 30\text{ = }\frac{15y+10y-20}{6} \\ 30(6)\text{ = 15y + 10y - 20} \\ 180\text{ = 25y - 20} \\ 180\text{ + 20 = 25y} \\ 25y\text{ = 200} \\ y\text{ = 200/25} \\ y\text{ = 8} \end{gathered}[/tex]

Substitute the value of y into equation (1)

[tex]\begin{gathered} 15\text{ + k = }\frac{5y}{2} \\ 15\text{ + k = }\frac{5(8)}{2} \\ 15\text{ + k = }\frac{40}{2} \\ 15\text{ + k = 20} \\ k\text{ = 20 - 15} \\ k\text{ = 5} \end{gathered}[/tex]

The rate of the current = 5 mph

What are the domain and range of y = cot x? Select onechoice for domain and one for range.

Answers

ANSWER:

A. Domain: x ≠ n

D. Range: All real numbers

STEP-BY-STEP EXPLANATION:

We have the following function:

[tex]y=\cot\left(x\right)[/tex]

The domain of a function is the interval of input values, that is, the interval of x while the range is the interval of output values, that is, the interval of y.

In the cotangent function, x cannot take the value of radians (nor its multiples), since it is not defined, while the range is continuous on all real numbers.

That means the correct options are:

A. Domain: x ≠ n

D. Range: All real numbers

what number is divisible by 5 ? 86,764,670,or27

Answers

The number divisible by 5 is 670.

Numbers divisible by 5 have their last digits as 0 or 5

Answer : 670

Can you please help me out with a question

Answers

right. the lateral area of a hemisfere is the curved area, wich is half the area of a complete sphere

area of a sphere:

4πr²

So, half the area is 1/2(4πr²)= 2πr²

Now, the total surface is the lateral area plus the area of the base. the base is a circle, so the area is equal to πr²

And the volume of a hemisfere is equal to half the volume of a sphere:

[tex](\frac{4}{3}\pi r^3)\cdot\frac{1}{2}\text{ =}\frac{2}{3}\pi r^3[/tex]

So, the anwsers are:

[tex]2\pi r^{2}\text{ = }2\pi(24ft)^{2}\text{ = 1152}\pi ft^2[/tex][tex]\pi r^{2}\text{ = }\pi(24ft)^2\text{ = 576}\pi ft^2[/tex][tex]\frac{2}{3}\pi r^3\text{ = }\frac{2}{3}\pi(24ft)^3\text{ = 9216}\pi ft^3[/tex]

The answers are in order

Today's previewYou can solve this by rearranging to create asituation to use the method from the previouslesson, or you can solve this by thinking a littledifferently about how the variables below mightalso be described.... so solve it.y = 2x + 4x + y = 7

Answers

[tex]\begin{gathered} y-2x=4 \\ y+x=7 \\ -2x-x=4-7 \\ -3x=-3 \\ x=3 \\ \\ y+x=7 \\ y=7-3 \\ y=4 \end{gathered}[/tex]

find the first term when the 31st 32nd and 33rd are 1.40, 1.55, and 1.70

Answers

jadeymae06, this is the solution:

This is an arithmetic sequence, where d (common difference) = 0.15

(1.70 - 1.55) or (1.55 - 1.40)

,

• a + 30d = 1.40

,

• a + 30(0.15) = 1.4

,

• a + 4.5 = 1.4

,

• a = 1.4 - 4.5

,

• a = -3.1

Jade, the first term is -3.1

PLEASE ITS URGENT I NEED HELP!!! I BEG YOU GUYS PLEEAASEEE THANKS..

Answers

[tex]b,c\text{ and d}[/tex]

Explanation

remember some properties of the exponents

[tex]\begin{gathered} a^m\cdot a^n=a^{m+n} \\ (a^m)^n=a^{m\cdot n} \\ a^{-m}=\frac{1}{a^m} \end{gathered}[/tex]

then, to solve this solve each option and compare

Step 1

[tex]6^{-5}\cdot6^2[/tex]

solve

[tex]\begin{gathered} 6^{-5}\cdot6^2=6^{-5+2}=6^{-3} \\ \end{gathered}[/tex]

so, this is not an answer

Step 2

[tex](\frac{1}{6^2})^5[/tex]

solve

[tex]\begin{gathered} (\frac{1}{6^2})^5=(6^{-2})^5=6^{(-2\cdot5)}=6^{-10} \\ \end{gathered}[/tex]

so, this is an answer

Step 3

[tex]\begin{gathered} (6^{-5})^2 \\ \text{solve} \\ (6^{-5})^2=6^{-5\cdot2}=6^{-10} \end{gathered}[/tex]

so, this is an answer

Step 4

[tex]\begin{gathered} \frac{6^{-3}}{6^7} \\ \text{solve} \\ \frac{6^{-3}}{6^7}=\frac{1}{6^3\cdot6^7}=\frac{1}{6^{3+7}}=\frac{1}{6^{10}}=6^{-10} \end{gathered}[/tex]

so, this is an answer

Step 5

[tex]\begin{gathered} \frac{6^5\cdot6^{-3}}{6^{-8}} \\ \text{solve} \\ \frac{6^5\cdot6^{-3}}{6^{-8}}=\frac{6^{5-3}}{6^{-8}}=\frac{6^2}{6^{-8}}=6^2\cdot\frac{1}{6^{-8}}=6^2\cdot6^8=6^{10} \end{gathered}[/tex]

so, this is not an answer

I hope this helps you

Other Questions
write a letter to your friend from britain that never visited Saudi arabia before the input redirection symbol, A cake is cut into 12 equal slices. After 3 days Jake has eaten 5 slices. What is his weekly rate of eating the cake?5363536cakes/weekcakes/week1 cakes/week3501 cakes/week How did being stereotyped negatively affect Joseph Suina? provide a specific example and briefly explain. Question 6:A particle of mass 0.2 kg is moving in a straight line at speed 2.5ms, a constant force of magnitude FN is applied to it in the direction in which it is moving. The speed of the particle 5 seconds later is 4.5ms Find the value of F, magnitude of the force applied. For each level of confidence o below, determine the corresponding normal confidence interval. Assume each confidence interval is constructed for the same sample statistics.Drag each normal confidence interval given above to the level of confidence A car with a weight of 6996 N is subjected to a net force of 1.74e4. The cars acceleration will be What is the relationship between n, , and the total number of nodes?The number of planar nodes = .The number of spherical nodes = n - - 1. jon's bathtub is rectangular and its base is 18 ft2. how fast is the water level rising if jon is filling the tub at a rate of 0.6 ft3/min? (use decimal notation. give your answer to three decimal places.) What happens when you cover burning candles with different size glass containers? You are the administrator of a 250-bed hospital. A recent report from the county tells you that the population within a 25-radius of your facility is getting younger - the median age in the area has decreased from 35 years of age to 29 years of age. This was an expected impact of the new state university campus that has opened about five miles away.a. What do you need to review and analyze to ensure that you can provide the services required by this changing population in your area?b. Are there any departments in your hospital that may need to be added or expanded? If so, specifically, which services?c. Are there any departments you might be able to reduce or eliminate? If so, specifically, which services?d. What details about clinical staff and administrative staff do you need to take into consideration for any of these changes? a client is looking for a very safe instrument that will not produce any current income but will provide a payout in ten years, providing funds that can then be deployed towards an educational goal. what is the most appropriate choice for this client? 2x-11x-21=0.using factorization method gabriel recently signed up for a website that reunites one with former classmates from elementary school. he is curious to see if many of his former classmates still look and act the same. research on stability and change would predict: In the plot cruel tribute which events are a result of king Minos actions ? Select 3 options Triangle FGH is similar to triangle IJK. Find the measure of side JK. Round youranswer to the nearest tenth if necessary. Suppose you stand with one foot on ceramic flooring and one foot on a wool carpet, making contact over an area of 80.0cm2 with each foot. Both the ceramic and the carpet are 2.00 cm thick and are 10.0C on their bottom sides. At what rate must heat transfer occurs from each foot to keep the top of the ceramic and carpet at 33.0C? Frankie was selected as the commencement speaker for this years graduation and experiencing imposter syndrome. To help overcome their imposter syndrome, frankie begins to engage in positive self-talk. Before frankie practices the speech, they now say that they are an amazing speaker and have a lot of great ideas to share. What practice is frankie engaging to overcome imposter syndrome?. True or false? The period of an AC supply is equal to the time between an adjacent peak and trough on an oscilloscope trace. business has increased such that a few years ago purple reign brought warehousing and distribution in-house to package and ship customer orders. with business continuing to expand, the company now is making the decision to continue to order boxes from box manufacturers or buy a corrugate machine that will manufacturer boxes to customized sizes on demand. the company expects to have 120,000 orders in 2022 and each order will use one box. when buying boxes from manufacturers, the company needs to buy different size boxes to account for different orders. this pushes the average cost of each box to $0.47. the box machine purple reign is considering costs $30,000, but lowers the variable cost of each box to $0.22 because the company is able to manufacture each box on-demand. if the company buys boxes from manufacturers, purple reign likes to have a contract with each to ensure needs are met to certain specifications. purple reign can have all contracts negotiated for $6,000. based on this information, and considering only the information we have based on 2022 projects, should purple reign purchase a box machine and make boxes on demand or should the company continue to buy boxes from outside providers? for 2022, what is the best economic decision?